Distribución limitante de

8

Dejar (Xn) ser una secuencia de iid N(0,1)variables aleatorias. DefinirS0=0 y Sn=k=1nXk para n1. Encuentre la distribución limitante de

1nk=1n|Sk1|(Xk21)

Este problema es de un libro de problemas sobre Teoría de la probabilidad, en el capítulo sobre el Teorema del límite central.

Ya que Sk1 y Xk son independientes E(|Sk1|(Xk21))=0 y

V(|Sk1|(Xk21))=E(Sk12(Xk21)2)=E(Sk12)E(Xk21)2)=2(k1)

Tenga en cuenta que el |Sk1|(Xk21)claramente no son independientes. El problema es de Problemas de probabilidad de Shiryaev , que se basa en el libro de texto del mismo autor. El libro de texto no parece cubrir el CLT para variables correlacionadas. No sé si hay una secuencia de mezcla estacionaria escondida en alguna parte ...

He realizado simulaciones para tener una idea de la respuesta.

import numpy as np
import scipy as sc
import scipy.stats as stats
import matplotlib.pyplot as plt

n = 20000 #summation index
m = 2000 #number of samples

X = np.random.normal(size=(m,n))
sums = np.cumsum(X, axis=1)
sums = np.delete(sums, -1, 1)
prods = np.delete(X**2-1, 0, 1)*np.abs(sums)
samples = 1/n*np.sum(prods, axis=1)

plt.hist(samples, bins=100, density=True)
x = np.linspace(-6, 6, 100)
plt.plot(x, stats.norm.pdf(x, 0, 1/np.sqrt(2*np.pi)))
plt.show()

A continuación se muestra un histograma de 2000 muestras (n=20.000) Parece distribuido con bastante normalidad ...

ingrese la descripción de la imagen aquí

Gabriel Romon
fuente
@MartijnWeterings Publiqué esto porque he reflexionado sobre el problema durante algún tiempo y estoy atascado. Probablemente esté lejos de ser trivial ...
Gabriel Romon
@MartijnWeterings E(|Sk1|(Xk21))=0por lo tanto V(|Sk1|(Xk21))=E(Sk12(Xk21)2)
Gabriel Romon
@MartijnWeterings Sí, omití la igualdad trivial |x|2=x2 para xR...
Gabriel Romon
El histograma en la simulación es una coincidencia terrible para la distribución Normal. Si no está convencido, calcule la curtosis.
whuber
@MartijnWeterings Sí, hice una omisión vergonzosa en el código. Lo actualicé, así como el histograma, que parece normal. ¿Tiene una idea del valor exacto de la varianza?
Gabriel Romon

Respuestas:

1

Cuando simulo la distribución, obtengo algo parecido a una distribución de Laplace. Aún mejor parece ser un q-gausiano (los parámetros exactos que tendrías que encontrar usando la teoría).

Supongo que su libro debe contener alguna variación del CLT que se relacione con eso (teorema del límite central generalizado q, probablemente esté en la Sección 7.6 El teorema del límite central para sumas de variables dependientes , pero no puedo buscarlo ya que no tengo el libro disponible).

simulación

library(qGaussian)
set.seed(1)
Qstore <- c(0) # vector to store result

n <- 10^6  # columns X_i
m <- 10^2  # rows repetitions

pb <- txtProgressBar(title = "progress bar", min = 0,
                     max = 100, style=3)
for (i in 1:100) {  
  # doing this several times because this matrix method takes a lot of memory
  # with smaller numbers n*m it can be done at once

  X <- matrix(rnorm(n*m,0,1),m)
  S <- t(sapply(1:m, FUN = function(x) cumsum(X[x,])))
  S <- cbind(rep(0,m),S[,-n])
  R <- abs(S)*(X^2-1)
  Q <- t(sapply(1:m, FUN = function(x) cumsum(R[x,])))

  Qstore <- c(Qstore,t(Q[,n]))
  setTxtProgressBar(pb, i)
}
close(pb)

# compute histogram 
x <- seq(floor(min(Qstore/n)), ceiling(max(Qstore/n)), 0.2)
h <- hist(Qstore/(n),breaks = x)

# plot simulation
plot( h$mid, h$density, log = "y", xlim=c(-7,7),
      ylab = "log density" , xlab = expression(over(1,n)*sum(abs(S[k-1])*(X[k]^2-1),k==1,n) ) )

# distributions for comparison
lines(x, dnorm(x,0,1),                   col=1, lty=3)      #normal 
lines(x, dexp(abs(x),sqrt(2))/2,         col=1, lty=2)      #laplace
lines(x, qGaussian::dqgauss(x,sqrt(2),0,1/sqrt(2)), col=1, lty=1)      #qgauss

# further plotting
title("10^4 repetitions with n=10^6")
legend(-7,0.6,c("Gaussian", "Laplace", "Q-Gaussian"),col=1, lty=c(3,2,1),cex=0.8)
Sexto Empírico
fuente
Con respecto al contenido del libro de texto, es mejor que lo vea por usted mismo: Volumen 1 , Volumen 2 . El problema solo debería requerir material cubierto en el Capítulo 3.4
Gabriel Romon
@GabrielRomon muchas gracias por esos enlaces. Mirándolo, desde mi teléfono, no pude encontrar nada sobre las distribuciones limitantes q-gaussianas u otras que no son una distribución normal. Entonces, o la distribución tiene una convergencia muy lenta n >> 10 ^ 6 antes de que la veamos , o la pregunta no se ajusta al capítulo (¿es del libro, tampoco pude encontrar la pregunta?). Una gráfica de los momentos de orden superior (como función de n) podría mostrar mejor si la conversión aún puede ocurrir, pero supongo que este no es un caso típico de CLT.
Sextus Empiricus
1
Este es el problema 3.4.14 en el libro de problemas .
Gabriel Romon